← Back to Topics

cardiology

Practice targeted AMC-style multiple-choice questions on cardiology.

Related Topics

A 60-year-old man presents to the emergency department with sudden onset of severe chest pain radiating to his back. He describes the pain as tearing in nature. His blood pressure is 180/100 mmHg in the right arm and 160/90 mmHg in the left arm. On examination, there is a new diastolic murmur. What is the most appropriate initial investigation?

A. Echocardiogram
B. D-dimer test
C. CT angiography of the chest
D. Electrocardiogram (ECG)
E. Chest X-ray
Mark this as a high-quality question
Mark this as a poor-quality or problematic question

A 65-year-old man with hypertension is started on a medication that blocks the angiotensin-converting enzyme (ACE). What is the most common side effect of this class of drugs?

A. Hyperkalemia
B. Acute kidney injury
C. Angioedema
D. Hypotension
E. Dry cough
Mark this as a high-quality question
Mark this as a poor-quality or problematic question

A 67-year-old man with a history of hypertension and type 2 diabetes is scheduled for elective cholecystectomy due to symptomatic gallstones. During the preoperative assessment, he mentions experiencing mild chest discomfort during exertion over the past few weeks. What is the most appropriate next step in his preoperative management?

A. Start the patient on aspirin and beta-blockers immediately
B. Order a preoperative chest X-ray
C. Refer for a cardiology evaluation and possible stress testing
D. Proceed with surgery as planned with close intraoperative monitoring
E. Schedule an urgent coronary angiogram
Mark this as a high-quality question
Mark this as a poor-quality or problematic question

A 58-year-old man presents to the emergency department with chest pain radiating to his left arm. His ECG shows ST-segment elevation in leads II, III, and aVF. What is the most appropriate immediate management?

A. Administer thrombolytic therapy
B. Perform an echocardiogram to assess cardiac function
C. Administer sublingual nitroglycerin and observe
D. Start intravenous heparin and admit to the coronary care unit
E. Administer aspirin and initiate primary percutaneous coronary intervention (PCI)
Mark this as a high-quality question
Mark this as a poor-quality or problematic question

A 65-year-old male with a history of hypertension and type 2 diabetes presents with stable angina. He is currently on metformin, atorvastatin, and lisinopril. Which of the following medications is most appropriate to add to his regimen to reduce angina symptoms?

A. Diuretic (e.g., hydrochlorothiazide)
B. Beta-blocker (e.g., metoprolol)
C. Angiotensin II receptor blocker (e.g., losartan)
D. Calcium channel blocker (e.g., amlodipine)
E. Nitrate (e.g., isosorbide mononitrate)
Mark this as a high-quality question
Mark this as a poor-quality or problematic question

A 72-year-old man with fatigue and a past myocardial infarction at 55 is breathless on stairs, has a 3 kg weight gain, and ankle marks. What is his likely NYHA functional class for chronic heart failure?

A. NYHA Class 3
B. NYHA Class 2
C. NYHA Class 4
D. NYHA Class 1
E. Not enough information to classify.
Mark this as a high-quality question
Mark this as a poor-quality or problematic question

A 25-year-old Aboriginal man presents to a remote clinic with increasing shortness of breath on exertion and palpitations over the past month. He recalls having a sore throat as a child but no specific diagnosis. On examination, his pulse is 95 bpm, blood pressure 110/70 mmHg, respiratory rate 20 breaths/min. Auscultation reveals a pansystolic murmur loudest at the apex, radiating to the axilla. There are no signs of heart failure currently. Considering the likely diagnosis and the patient's background, which of the following is the most appropriate initial investigation?

A. D-dimer
B. Cardiac troponin
C. Electrocardiogram (ECG)
D. Chest X-ray
E. Echocardiogram
Mark this as a high-quality question
Mark this as a poor-quality or problematic question

A 68-year-old male with a history of hypertension and stable angina is prescribed amlodipine. He returns after two weeks complaining of significant ankle edema. What is the most appropriate initial management step?

A. Add a beta-blocker to counteract the effects of amlodipine.
B. Advise the patient to elevate his legs and wear compression stockings.
C. Reduce the dose of amlodipine.
D. Switch to an alternative antihypertensive such as an ACE inhibitor or ARB.
E. Prescribe a diuretic to manage the edema.
Mark this as a high-quality question
Mark this as a poor-quality or problematic question

A 58-year-old man with a history of hypertension and left ventricular systolic dysfunction (LVSD) is being treated with an ACE inhibitor. His blood pressure is well-controlled, but he develops a persistent, dry cough that is affecting his sleep and quality of life. What is the most appropriate next step in management?

A. Add a cough suppressant
B. Switch to an angiotensin II receptor blocker (ARB)
C. Add a diuretic
D. Discontinue the ACE inhibitor and start a beta-blocker
E. Reduce the dose of the ACE inhibitor
Mark this as a high-quality question
Mark this as a poor-quality or problematic question

A 70-year-old man with a history of hypertension and hyperlipidemia presents to the emergency department with sudden onset, severe chest pain radiating to his back. He is diaphoretic and anxious. His blood pressure is 90/60 mmHg. Which of the following is the most likely diagnosis?

A. Esophageal rupture
B. Pulmonary embolism
C. Acute myocardial infarction
D. Pericarditis
E. Aortic dissection
Mark this as a high-quality question
Mark this as a poor-quality or problematic question

A 72-year-old man with a history of type 2 diabetes mellitus, hypertension, and chronic kidney disease (stage 3) presents with a 2-week history of exertional dyspnea and fatigue. He reports no chest pain but has noticed occasional palpitations. On examination, his blood pressure is 150/90 mmHg, heart rate is 110 bpm (irregularly irregular), respiratory rate is 20 breaths per minute, and oxygen saturation is 94% on room air. Cardiovascular examination reveals a variable intensity S1, no S3 or S4, and a soft systolic murmur at the apex. Lung fields are clear. An ECG shows absent P waves and irregularly irregular QRS complexes. Which of the following is the most appropriate next step in management?

A. Start a beta-blocker for rate control
B. Administer intravenous digoxin
C. Perform immediate electrical cardioversion
D. Initiate anticoagulation therapy
E. Refer for urgent coronary angiography
Mark this as a high-quality question
Mark this as a poor-quality or problematic question
A contrast enhanced CT scan demonstrating an abdominal aortic aneurysm of 4.8 * 3.8 cm
Image by James Heilman, MD CC BY-SA 3.0 · Source

A 68-year-old male presents for routine follow-up. He is asymptomatic. Review the image. What is the most appropriate next step in management?

A. Commence dual antiplatelet therapy
B. Repeat imaging in 6-12 months
C. Prescribe a statin
D. Initiate beta-blocker therapy
E. Schedule elective surgical repair
Mark this as a high-quality question
Mark this as a poor-quality or problematic question

A 68-year-old woman with a history of hypertension and heart failure is admitted to the hospital with increasing shortness of breath and peripheral edema. She is currently taking furosemide 40mg daily. Her blood tests reveal the following: Na+ 130 mmol/L (135-145 mmol/L), K+ 3.1 mmol/L (3.5-5.0 mmol/L), Cl- 95 mmol/L (98-107 mmol/L), HCO3- 32 mmol/L (22-29 mmol/L). What is the most appropriate initial management?

A. Administer intravenous normal saline and continue furosemide at the same dose.
B. Administer an ACE inhibitor and monitor sodium levels.
C. Administer intravenous sodium bicarbonate and increase the dose of furosemide.
D. Hold furosemide, administer intravenous potassium chloride, and restrict free water intake.
E. Administer intravenous calcium gluconate and monitor potassium levels.
Mark this as a high-quality question
Mark this as a poor-quality or problematic question

A 45-year-old male with a history of hypertension presents with recurrent episodes of palpitations and near-syncope. His ECG during an episode shows a wide QRS complex tachycardia with a rate of 220 bpm. There is no evidence of structural heart disease on echocardiography. Which of the following is the most likely underlying mechanism of his arrhythmia?

A. Triggered activity due to early afterdepolarizations
B. Re-entry within the Purkinje system
C. Abnormal impulse conduction through the atrioventricular node
D. Enhanced automaticity in the sinoatrial node
E. Myocardial ischemia leading to ventricular tachycardia
Mark this as a high-quality question
Mark this as a poor-quality or problematic question

A 72-year-old man with a history of chronic heart failure with reduced ejection fraction (HFrEF) and atrial fibrillation presents to the cardiology clinic for routine follow-up. He reports increased fatigue and occasional palpitations but denies chest pain or syncope. His current medications include lisinopril, carvedilol, furosemide, and digoxin. His blood pressure is 110/70 mmHg, heart rate is 78 bpm, and he has mild peripheral edema. Recent laboratory tests show a serum creatinine of 1.2 mg/dL and potassium of 4.5 mmol/L. An echocardiogram shows an ejection fraction of 35%. Considering his current medication regimen and clinical status, which of the following is the most appropriate next step in his pharmacological management?

A. Increase the dose of carvedilol
B. Initiate ivabradine
C. Start warfarin for anticoagulation
D. Switch lisinopril to sacubitril/valsartan
E. Add spironolactone
Mark this as a high-quality question
Mark this as a poor-quality or problematic question

A 68-year-old woman with a history of heart failure presents to the emergency department with increasing shortness of breath and lower extremity edema. She has been taking furosemide 40mg daily for the past year. Her blood pressure is 110/70 mmHg, heart rate is 90 bpm, and respiratory rate is 24 breaths/min. An ECG shows flattened T waves and prominent U waves. Which of the following is the most likely electrolyte abnormality contributing to her presentation?

A. Hypomagnesemia
B. Hypercalcemia
C. Hyperkalemia
D. Hypokalemia
E. Hyponatremia
Mark this as a high-quality question
Mark this as a poor-quality or problematic question

What is the most common cardiac anomaly associated with Noonan syndrome?

A. Atrial septal defect
B. Aortic stenosis
C. Pulmonary stenosis
D. Coarctation of the aorta
E. Ventricular septal defect
Mark this as a high-quality question
Mark this as a poor-quality or problematic question
X-ray of hiatal hernia
Image by Ahmed Farhat, Daryn Towle CC BY 4.0 · Source

An 82-year-old female presents with a 3-month history of intermittent retrosternal burning pain, worse after meals, and occasional shortness of breath on exertion. Vital signs are stable. Physical exam unremarkable. A chest X-ray is performed. Considering the findings on the image, what is the most appropriate initial management step?

A. Reassure the patient that the finding is benign and requires no further action.
B. Urgent surgical consultation for repair.
C. Initiate a trial of proton pump inhibitor therapy.
D. Proceed directly to oesophago-gastro-duodenoscopy (OGD).
E. Refer for a stress echocardiogram to rule out cardiac ischaemia.
Mark this as a high-quality question
Mark this as a poor-quality or problematic question

Which zoonotic infection is a leading cause of culture-negative endocarditis?

A. Bartonellosis
B. Q fever
C. Brucellosis
D. Leptospirosis
E. Psittacosis
Mark this as a high-quality question
Mark this as a poor-quality or problematic question

Reciprocal ST-depression with ST-elevation in leads II, III, aVF is most likely seen in which leads?

A. V4-V6
B. V7-V9
C. aVL and Lead I
D. V1-V3
E. aVR
Mark this as a high-quality question
Mark this as a poor-quality or problematic question

A 60-year-old man presents to the emergency department with sudden onset of severe chest pain that radiates to his back. He describes the pain as tearing in nature. His blood pressure is 180/100 mmHg in the right arm and 160/90 mmHg in the left arm. On examination, there is a new diastolic murmur heard best at the right sternal border. What is the most appropriate next step in management?

A. Administer sublingual nitroglycerin
B. Perform an ECG
C. Order a CT angiography of the chest
D. Administer aspirin
E. Start intravenous heparin
Mark this as a high-quality question
Mark this as a poor-quality or problematic question

A 72-year-old man with a history of heart failure and hypertension is admitted to the hospital with pneumonia. His medications include furosemide, ramipril, and digoxin. On day 3 of admission, he develops new-onset polymorphic ventricular tachycardia (Torsades de Pointes). An ECG shows a QTc interval of 580 ms. Which of the following is the MOST likely contributing factor to his arrhythmia?

A. Pneumonia-related hypoxia
B. Digoxin toxicity
C. Underlying structural heart disease
D. Ramipril-induced bradycardia
E. Electrolyte imbalance secondary to furosemide
Mark this as a high-quality question
Mark this as a poor-quality or problematic question

A 62-year-old man presents to the emergency department with acute onset chest pain radiating to his left arm and jaw, accompanied by diaphoresis and nausea. He has a history of hypertension, type 2 diabetes mellitus, and hyperlipidemia. On examination, his blood pressure is 160/95 mmHg, heart rate is 95 bpm, respiratory rate is 20 breaths per minute, and oxygen saturation is 96% on room air. An ECG shows ST-segment elevation in leads II, III, and aVF. Initial troponin levels are elevated. He is given aspirin and clopidogrel in the emergency department. Which of the following is the most appropriate next step in management?

A. Urgent percutaneous coronary intervention (PCI)
B. Intravenous nitroglycerin
C. Coronary artery bypass grafting (CABG)
D. Intravenous thrombolysis
E. Beta-blocker administration
Mark this as a high-quality question
Mark this as a poor-quality or problematic question

A 70-year-old man with known atrial fibrillation presents with sudden onset of severe pain, pallor, and coldness in his left leg. Pulses are absent below the femoral artery. What is the most likely source of the embolus causing this presentation?

A. Left atrium
B. Deep vein thrombosis
C. Popliteal artery aneurysm
D. Aortic arch
E. Carotid artery
Mark this as a high-quality question
Mark this as a poor-quality or problematic question
X-ray of hiatal hernia
Image by Ahmed Farhat, Daryn Towle CC BY 4.0 · Source

A 72-year-old male presents with increasing shortness of breath and epigastric discomfort, particularly after meals. He has a history of well-controlled hypertension. An X-ray is performed. What is the MOST appropriate next step in management?

A. Perform an upper endoscopy
B. Refer for surgical consultation for fundoplication
C. Initiate a proton pump inhibitor and lifestyle modifications
D. Prescribe a prokinetic agent such as metoclopramide
E. Order a barium swallow study
Mark this as a high-quality question
Mark this as a poor-quality or problematic question

A 65-year-old man with a history of hypertension and ischemic heart disease presents to the emergency department with worsening shortness of breath, orthopnea, and peripheral edema. On examination, he has elevated jugular venous pressure, bilateral basal crackles, and pitting edema in the lower limbs. An echocardiogram shows an ejection fraction of 35%. What is the most appropriate initial management step for this patient?

A. Initiate digoxin therapy
B. Perform coronary angiography
C. Start oral beta-blockers
D. Administer intravenous furosemide
E. Prescribe oral ACE inhibitors
Mark this as a high-quality question
Mark this as a poor-quality or problematic question

A 65-year-old man with a history of type 2 diabetes mellitus and hypertension presents with exertional dyspnea and fatigue. His physical examination reveals a blood pressure of 150/90 mmHg, a heart rate of 88 bpm, and a soft S4 gallop. There is no peripheral edema. An electrocardiogram shows left ventricular hypertrophy with repolarization abnormalities. Which of the following diagnostic tests would be most appropriate to evaluate for underlying coronary artery disease in this patient?

A. Coronary angiography
B. Cardiac MRI
C. 24-hour Holter monitoring
D. Transthoracic echocardiography without stress
E. Exercise stress echocardiography
Mark this as a high-quality question
Mark this as a poor-quality or problematic question

A 68-year-old man with a history of hypertension, type 2 diabetes, and stable angina is scheduled for elective hernia repair surgery. He is currently on aspirin, metformin, lisinopril, and atorvastatin. His last angina episode was over a year ago, and he has no history of myocardial infarction. What is the most appropriate preoperative management step to minimize his cardiovascular risk during surgery?

A. Discontinue aspirin therapy one week before surgery
B. Increase the dose of atorvastatin
C. Continue aspirin therapy
D. Order a preoperative stress test
E. Start beta-blocker therapy
Mark this as a high-quality question
Mark this as a poor-quality or problematic question

A 72-year-old woman is admitted to the cardiology ward following an episode of syncope. Her past medical history includes hypertension, for which she takes hydrochlorothiazide, and osteoarthritis, managed with regular ibuprofen. An ECG reveals a prolonged QTc interval of 520 ms. She denies any family history of sudden cardiac death or prolonged QT syndrome. Her electrolytes are within normal limits except for a potassium level of 3.4 mmol/L (normal range 3.5-5.0 mmol/L). Which of the following is the MOST appropriate initial step in managing this patient?

A. Administer intravenous magnesium sulfate.
B. Initiate treatment with a beta-blocker such as metoprolol.
C. Commence amiodarone therapy.
D. Insert an implantable cardioverter-defibrillator (ICD).
E. Correct the hypokalemia and discontinue hydrochlorothiazide.
Mark this as a high-quality question
Mark this as a poor-quality or problematic question

A 68-year-old man with a history of hypertension and coronary artery disease presents to the emergency department with worsening shortness of breath and fatigue over the past week. On examination, he has elevated jugular venous pressure, bilateral lung crackles, and peripheral edema. An echocardiogram reveals reduced ejection fraction and dilated ventricles. Which of the following best explains the pathophysiological mechanism leading to his symptoms?

A. Increased peripheral resistance leading to left ventricular hypertrophy
B. Decreased venous return causing systemic hypotension
C. Enhanced myocardial contractility resulting in fluid retention
D. Decreased cardiac output leading to activation of the renin-angiotensin-aldosterone system
E. Increased cardiac output causing pulmonary congestion
Mark this as a high-quality question
Mark this as a poor-quality or problematic question

A 45-year-old man with a history of hypertension presents to the emergency department with palpitations and lightheadedness. His ECG shows a regular narrow-complex tachycardia at a rate of 180 bpm. There are no visible P waves, and the QRS duration is 0.08 seconds. He is hemodynamically stable. Which of the following is the most appropriate initial management?

A. Synchronized cardioversion
B. Observation without intervention
C. Intravenous adenosine
D. Vagal maneuvers
E. Intravenous metoprolol
Mark this as a high-quality question
Mark this as a poor-quality or problematic question

A 72-year-old man with a history of hypertension and atrial fibrillation presents to the emergency department with worsening shortness of breath and fatigue over the past two weeks. He reports occasional palpitations but denies chest pain or syncope. On examination, his blood pressure is 110/70 mmHg, heart rate is 130 bpm (irregularly irregular), respiratory rate is 22 breaths per minute, and oxygen saturation is 94% on room air. Jugular venous pressure is elevated, and there are bilateral lung crackles. An ECG shows atrial fibrillation with a rapid ventricular response. An echocardiogram reveals moderate mitral regurgitation and left atrial enlargement. Which of the following is the most appropriate initial management step?

A. Initiation of antiarrhythmic therapy with amiodarone
B. Rate control with intravenous diltiazem
C. Immediate electrical cardioversion
D. Initiation of oral anticoagulation
E. Referral for mitral valve surgery
Mark this as a high-quality question
Mark this as a poor-quality or problematic question

A 16-year-old Aboriginal boy from a remote community presents to the local clinic with a two-week history of increasing fatigue, occasional shortness of breath on exertion, and migratory joint pains affecting his knees and ankles. He had a documented episode of acute rheumatic fever (ARF) three years ago, complicated by mild mitral regurgitation, and was commenced on monthly benzathine penicillin G injections for secondary prophylaxis. However, his adherence has been inconsistent over the past year due to difficulties accessing the clinic. On examination, he is afebrile. His pulse is 95 bpm, blood pressure 110/70 mmHg, and respiratory rate 18 breaths/min. Cardiac auscultation reveals a soft apical pansystolic murmur, unchanged from his last review six months ago. There is mild swelling and tenderness in his left ankle joint, but no erythema or warmth. His throat swab for *Streptococcus pyogenes* is negative. Laboratory tests show a CRP of 45 mg/L (normal <5), ESR 60 mm/hr (normal <15), and a normal full blood count. An ECG shows sinus rhythm with no PR interval prolongation. A point-of-care ultrasound shows mild mitral regurgitation. Considering the clinical presentation, history, and the significant burden of rheumatic heart disease in this population, which of the following is the most appropriate immediate management step regarding his secondary prophylaxis?

A. Re-establish and reinforce the importance of regular monthly benzathine penicillin G injections.
B. Increase the dose of benzathine penicillin G to fortnightly injections.
C. Initiate a course of oral corticosteroids for suspected recurrent ARF.
D. Arrange urgent formal echocardiogram to assess for progression of valvular disease.
E. Switch secondary prophylaxis to oral penicillin V daily.
Mark this as a high-quality question
Mark this as a poor-quality or problematic question

A 55-year-old man with a history of hypertension and smoking presents for a routine check-up. He is concerned about his risk of cardiovascular disease and asks for advice on lifestyle modifications. Which of the following lifestyle changes would most significantly reduce his risk of cardiovascular events?

A. Adopting a Mediterranean diet
B. Smoking cessation
C. Increasing physical activity
D. Limiting alcohol consumption
E. Reducing dietary salt intake
Mark this as a high-quality question
Mark this as a poor-quality or problematic question

A 55-year-old male with a history of hypertension presents for a routine check-up. His blood pressure is consistently measured at 150/95 mmHg despite lifestyle modifications. According to Australian guidelines, what is the most appropriate next step in managing his hypertension?

A. Prescribe a diuretic
B. Increase lifestyle modifications
C. Refer to a cardiologist
D. Start a beta-blocker
E. Initiate an ACE inhibitor
Mark this as a high-quality question
Mark this as a poor-quality or problematic question

A 72-year-old man with fatigue, a past myocardial infarction at age 55, breathlessness on stairs, and ankle marks has an LVEF < 40% on echocardiogram. Which of the following medications is known to improve outcomes in Heart Failure with Reduced Ejection Fraction (HFrEF)?

A. Digoxin
B. Verapamil
C. Frusemide
D. Amlodipine
E. Eplerenone
Mark this as a high-quality question
Mark this as a poor-quality or problematic question
X-ray of hiatal hernia
Image by Ahmed Farhat, Daryn Towle CC BY 4.0 · Source

An 82-year-old patient presents to their GP with a 3-month history of intermittent retrosternal chest discomfort, often described as a burning sensation, worse after meals and when lying flat. They also report mild shortness of breath on exertion and occasional regurgitation, but deny significant dysphagia or weight loss. Past medical history includes hypertension and osteoarthritis. Medications are perindopril and paracetamol. On examination, vital signs are stable (BP 130/80, HR 75, RR 16, SpO2 97% on air), and lung sounds are clear. A chest X-ray is performed as part of the workup. Based on the clinical context and the findings demonstrated in the image, what is the most appropriate next step in investigation?

A. Reassure the patient that the X-ray is normal and advise lifestyle modifications
B. Oesophago-gastro-duodenoscopy (OGD)
C. Refer for formal cardiac stress testing
D. Commence a trial of high-dose proton pump inhibitor (PPI) therapy
E. Perform pulmonary function tests (PFTs)
Mark this as a high-quality question
Mark this as a poor-quality or problematic question

An electrocardiogram (ECG) shows a supraventricular tachycardia (SVT). What is the first-line treatment?

A. Adenosine
B. Verapamil
C. Valsalva maneuver
D. Synchronized cardioversion.
E. Beta-blockers
Mark this as a high-quality question
Mark this as a poor-quality or problematic question

A 62-year-old man presents to the emergency department with a history of palpitations and dizziness that started suddenly 2 hours ago. He has a history of hypertension and is on lisinopril. He denies chest pain, shortness of breath, or syncope. On examination, his blood pressure is 110/70 mmHg, heart rate is 150 bpm (irregular), respiratory rate is 18 breaths per minute, and temperature is 36.7°C. An ECG is performed, showing an irregularly irregular rhythm with no distinct P waves and a narrow QRS complex. Which of the following is the most appropriate initial management step?

A. Initiation of anticoagulation therapy
B. Rate control with a beta-blocker
C. Immediate electrical cardioversion
D. Administration of adenosine
E. Rate control with digoxin
Mark this as a high-quality question
Mark this as a poor-quality or problematic question

A 45-year-old male presents to the emergency department with severe chest pain radiating to his left arm, shortness of breath, and diaphoresis. He has a history of hypertension and hyperlipidemia. On examination, he is diaphoretic, and his blood pressure is 90/60 mmHg. An ECG shows ST-segment elevation in leads II, III, and aVF. What is the most appropriate initial management for this patient?

A. Administer intravenous fluids to address hypotension
B. Administer beta-blockers to reduce myocardial oxygen demand
C. Administer morphine for pain management
D. Administer aspirin and initiate reperfusion therapy
E. Administer nitroglycerin for chest pain relief
Mark this as a high-quality question
Mark this as a poor-quality or problematic question

A 65-year-old patient with a history of DVT presents with sudden onset dyspnoea and pleuritic chest pain. Examination is unremarkable except for mild tachypnoea. Wells score is 4. Which initial investigation is most appropriate in the diagnostic pathway?

A. D-dimer
B. Lower limb ultrasound
C. V/Q scan
D. CT Pulmonary Angiogram (CTPA)
E. Chest X-ray
Mark this as a high-quality question
Mark this as a poor-quality or problematic question
X-ray of hiatal hernia
Image by Ahmed Farhat, Daryn Towle CC BY 4.0 · Source

A 72-year-old male presents with increasing shortness of breath and epigastric discomfort, particularly after meals. He has a history of well-controlled hypertension and takes aspirin daily. An X-ray is performed. What is the most appropriate next step in management?

A. Prescribe a H2 receptor antagonist
B. Order a barium swallow study
C. Initiate a proton pump inhibitor and lifestyle modifications
D. Perform an upper endoscopy
E. Refer for surgical consultation for fundoplication
Mark this as a high-quality question
Mark this as a poor-quality or problematic question

A 35-year-old patient with schizophrenia is on risperidone and develops QT prolongation on ECG. What is the next step?

A. Switch to clozapine.
B. Start a beta-blocker.
C. Discontinue risperidone.
D. Monitor ECG regularly.
E. Reduce the dose of risperidone.
Mark this as a high-quality question
Mark this as a poor-quality or problematic question

A middle-aged man who already has a 20% cardiovascular risk and is on lifestyle modification and statin therapy: Which one of the following helps you assess his cardiovascular risk?

A. Coronary artery angiogram
B. ECG
C. Exercise ECG
D. Echocardiogram
E. Coronary artery calcium score
Mark this as a high-quality question
Mark this as a poor-quality or problematic question

A 55-year-old male presents with chest pain that radiates to his left arm and jaw, accompanied by diaphoresis and nausea. ECG shows ST-segment elevation in leads II, III, and aVF. What is the most likely diagnosis?

A. Unstable angina
B. Acute inferior myocardial infarction
C. Acute anterior myocardial infarction
D. Pulmonary embolism
E. Aortic dissection
Mark this as a high-quality question
Mark this as a poor-quality or problematic question

A 55-year-old man with a history of hypertension presents for a routine check-up. His blood pressure is consistently measured at 160/95 mmHg despite lifestyle modifications. He is currently on perindopril 5 mg daily. According to Australian guidelines, what is the most appropriate next step in managing his hypertension?

A. Switch to losartan
B. Refer for renal artery imaging
C. Add hydrochlorothiazide
D. Add amlodipine
E. Increase perindopril to 10 mg daily
Mark this as a high-quality question
Mark this as a poor-quality or problematic question

A 62-year-old woman presents to the emergency department with acute onset of chest pain and shortness of breath. She describes the pain as sharp and worse with inspiration. She has a history of deep vein thrombosis and is currently on warfarin. On examination, her blood pressure is 110/70 mmHg, heart rate is 110 bpm, respiratory rate is 24 breaths per minute, and oxygen saturation is 88% on room air. Her jugular venous pressure is elevated, and she has clear lung fields on auscultation. An ECG shows sinus tachycardia with right axis deviation. Which of the following is the most likely diagnosis?

A. Acute myocardial infarction
B. Pulmonary embolism
C. Pericarditis
D. Pneumothorax
E. Aortic dissection
Mark this as a high-quality question
Mark this as a poor-quality or problematic question

A 62-year-old man presents to the emergency department with sudden onset chest pain and shortness of breath that started 2 hours ago while he was gardening. He describes the pain as a tightness across his chest, radiating to his left arm. He has a history of hypertension and hyperlipidemia, and he is a current smoker. On examination, his blood pressure is 150/90 mmHg, heart rate is 110 bpm, respiratory rate is 22 breaths per minute, and oxygen saturation is 94% on room air. He appears anxious and diaphoretic. An ECG shows ST-segment elevation in leads II, III, and aVF. Which of the following is the most appropriate immediate management step?

A. Administer intravenous morphine for pain relief
B. Administer beta-blockers to control heart rate
C. Administer oxygen to maintain saturation above 95%
D. Administer aspirin and initiate primary percutaneous coronary intervention (PCI)
E. Administer sublingual nitroglycerin and observe for response
Mark this as a high-quality question
Mark this as a poor-quality or problematic question
Right leg affected by warfarin necrosis
Image by Bakoyiannis C, Karaolanis G, Patelis N, Maskanakis A, Tsaples G, Klonaris C, Georgopoulos S, Liakakos T CC BY 4.0 · Source

A 75-year-old woman on warfarin for chronic atrial fibrillation presents to the emergency department with a 24-hour history of rapidly worsening pain and discolouration in her right lower leg. She denies trauma. Her INR is 2.8. Physical examination reveals the findings shown in the image. Peripheral pulses are palpable. What is the most appropriate immediate management step?

A. Perform urgent surgical debridement of the affected area
B. Order a Doppler ultrasound of the leg to rule out deep vein thrombosis
C. Stop warfarin and administer Vitamin K and heparin
D. Increase the dose of warfarin to achieve a higher INR
E. Administer broad-spectrum intravenous antibiotics for suspected cellulitis
Mark this as a high-quality question
Mark this as a poor-quality or problematic question

A middle-aged man with a 20% estimated cardiovascular risk is already on lifestyle modifications and statin therapy. Which of the following investigations can help further assess or reclassify his cardiovascular risk?

A. Coronary artery angiogram
B. Echocardiogram
C. Stress echocardiogram
D. Coronary artery calcium score
E. Exercise ECG
Mark this as a high-quality question
Mark this as a poor-quality or problematic question